3rd Grade Math Question

3rd Grade Math Question

Answers

Answer 1

Answer:

∠ 1 = 110° , ∠ 2 = 70°

Step-by-step explanation:

∠ 1 and ∠ 2 lie on a straight line and sum to 180° , that is

6x + 20 + 4x + 10 = 180

10x + 30 = 180 ( subtract 30 from both sides )

10x = 150 ( divide both sides by 10 )

x = 15

Then

∠ 1 = 6x + 20 = 6(15) + 20 = 90 + 20 = 110°

∠ 2 = 4x + 10 = 4(15) + 10 = 60 + 10 = 70°


Related Questions

The effectiveness of a blood-pressure drug is being investigated. An experimenter finds that, on average, the reduction in systolic blood pressure is 23.5 for a sample of size 775 and standard deviation 12.2. Estimate how much the drug will lower a typical patient's systolic blood pressure (using a 95% confidence level). Enter your answer as a tri-linear inequality accurate to one decimal place (because the sample statistics are reported accurate to one decimal place).

Answers

The 95% confidence interval for the effectiveness of the blood-pressure drug is given as follows:

[tex]22.6 < \mu < 24.4[/tex]

How to obtain the confidence interval?

The mean, the standard deviation and the sample size for this problem, which are the three parameters, are given as follows:

[tex]\overline{x} = 23.5, \sigma = 12.2, n = 775[/tex]

Looking at the z-table, the critical value for a 95% confidence interval is given as follows:

z = 1.96.

The lower bound of the interval is then given as follows:

[tex]23.5 - 1.96 \times \frac{12.2}{\sqrt{775}} = 22.6[/tex]

The upper bound of the interval is then given as follows:

[tex]23.5 + 1.96 \times \frac{12.2}{\sqrt{775}} = 24.4[/tex]

More can be learned about the z-distribution at https://brainly.com/question/25890103

#SPJ1

Roberto made a line plot to show the weight in pounds of the bags of granola in his store he concluded that the total weight of the granola was 2/1/2+2/3/4+3=8/1/4 pounds

Answers

The correct total weight of the bags of granola is 8 1/4 pounds.

One thing that can be done to improve Roberto's reasoning is to ensure the accuracy of the calculations.

In his conclusion, Roberto added the weights of the bags of granola (2 1/2, 2 3/4, and 3) and claimed that the total weight was 8 1/4 pounds. However, the sum of these weights does not equal 8 1/4 pounds.

To address this, Roberto should recheck his calculations. Adding mixed numbers involves adding the whole numbers separately and then adding the fractions separately. In this case, 2 1/2 + 2 3/4 + 3 can be calculated as follows:

2 + 2 + 3 = 7 (sum of whole numbers)

1/2 + 3/4 = 2/4 + 3/4 = 5/4 = 1 1/4 (sum of fractions)

Thus, the correct sum is 7 + 1 1/4 = 8 1/4 pounds.

By double-checking the calculations and providing the accurate sum, Roberto's reasoning would be more precise, reliable, and free from errors.

For more such questions on granola,click on

https://brainly.com/question/27741276

#SPJ8

The probable question may be:

Roberto made a line plot to show the weight in pounds of the bags of granola in his store he concluded that the total weight of the granola was 2 1/2+2 3/4+3=8 1/4 pounds.

what is one thing that you could do to Roberto's Reasoning

Tasha used the pattern in the table to find the value of 4 Superscript negative 4.

Powers of 4
Value
4 squared
16
4 Superscript 1
4
4 Superscript 0
1
4 Superscript negative 1
One-fourth
4 Superscript negative 2
StartFraction 1 Over 16 EndFraction

She used these steps.

Step 1 Find a pattern in the table.
The pattern is to divide the previous value by 4 when the exponent decreases by 1.
Step 2 Find the value of 4 Superscript negative 3.
4 Superscript negative 3 = StartFraction 1 Over 16 EndFraction divided by 4 = StartFraction 1 Over 16 EndFraction times one-fourth = StartFraction 1 Over 64 EndFraction
Step 3 Find the value of 4 Superscript negative 4.
4 Superscript negative 4 = StartFraction 1 Over 64 EndFraction divided by 4 = StartFraction 1 Over 64 EndFraction times one-fourth = StartFraction 1 Over 256 EndFraction
Step 4 Rewrite the value for 4 Superscript negative 4.
StartFraction 1 Over 256 EndFraction = negative StartFraction 1 Over 4 Superscript negative 4 EndFraction

Answers

The value of 4 Superscript negative 4 is negative StartFraction 1 Over 4 Superscript negative 4 EndFraction.

In the given table, Tasha observed a pattern in the powers of 4. When the exponent decreases by 1, the previous value is divided by 4. Using this pattern, she determined the values for 4 squared, 4 Superscript 1, 4 Superscript 0, 4 Superscript negative 1, and 4 Superscript negative 2.

To find the value of 4 Superscript negative 3, she divided the previous value (StartFraction 1 Over 16 EndFraction) by 4, resulting in StartFraction 1 Over 64 EndFraction.

Similarly, for 4 Superscript negative 4, she divided the previous value (StartFraction 1 Over 64 EndFraction) by 4, yielding StartFraction 1 Over 256 EndFraction.

Finally, to rewrite the value for 4 Superscript negative 4, she expressed it as negative StartFraction 1 Over 4 Superscript negative 4 EndFraction.

Therefore, the value of 4 Superscript negative 4 is negative StartFraction 1 Over 4 Superscript negative 4 EndFraction, which simplifies to StartFraction 1 Over 256 EndFraction

For more such answers on Superscript

https://brainly.com/question/29971585

#SPJ8

6. The diagram shows two points A and B. B is 40m East and then 55m South. Work out the bearing of B from A. ​

Answers

Answer:

X=68m

Step-by-step explanation:

X²=40²+55²

X²=1600+3025

X²=4625

X=(4625)½

X=68m.

60+40[tex]60+40[/tex]

Answers

60 + 40 = 100

100 is the answer

PLSSS HELP 13 POINTS

Answers

The equation of the line perpendicular to  y = 2 / 3 x - 4 and passes through (6, -2) is y = - 3 / 2x + 7.

How to represent equation in slope intercept form?

The equation of a line can be represented in slope intercept form as follows:

y = mx + b

where

m = slope of the lineb = y-intercept

The slopes of perpendicular lines are negative reciprocals of one another.

Therefore, the slope of the line perpendicular to y = 2 / 3 x - 4 is - 3 / 2.

Hence, let's find the line as its passes through (6, -2).

Therefore,

y = - 3 / 2 x + b

-2 = - 3 / 2(6) + b

-2  = -9 + b

b = -2 + 9

b = 7

Therefore, the equation of the line is y = - 3 / 2x + 7.

learn more on slope intercept form here: https://brainly.com/question/29146348

#SPJ1

foreign direct investment helps improve the economic situation of a recipient country by increasing —- opportunities in the country that the company invests in.

Answers

Foreign direct investment helps improve the economic situation of a recipient country by increasing employment opportunities in the country that the company invests in.

When foreign companies invest in a recipient country, they often establish or expand their operations, which requires hiring local workers. This leads to job creation and reduces unemployment rates in the recipient country.

Increased employment opportunities result in more individuals having access to income and improved standards of living.

Foreign direct investment also contributes to the transfer of technology, knowledge, and skills to the recipient country. Multinational companies often bring advanced technologies, production techniques, and management practices that may not have been available or widely adopted in the recipient country.

Furthermore, foreign direct investment stimulates domestic investment and encourages the growth of local businesses. When foreign companies invest in a recipient country, they often form partnerships or engage in supply chain relationships with local firms.

Overall, foreign direct investment increases employment opportunities, fosters technology transfer, and stimulates domestic investment, all of which contribute to improving the economic situation of a recipient country.

for similar questions on company invests.

https://brainly.com/question/27717275

#SPJ8

A popular restaurant has 48 tables. On each table are 3 different types of salsa. In one day, all of the tables are used for 9 different sets of customers. Which expression can be used to estimate how many containers of salsa are needed for all the tables in one day?

A 50 × 9
B 16 × 3 × 9
C 50 × 3 × 10
D 40 × 5 × 5

Answers

The expression to estimate the number of containers of salsa needed is: 48 × 3 × 9. none of the option is correct.

To estimate how many containers of salsa are needed for all the tables in one day, we need to consider the total number of tables and the number of salsa containers required for each table.

Given that there are 48 tables and each table has 3 different types of salsa, we can estimate the total number of containers needed by multiplying the number of tables by the number of salsa types.

However, we also need to account for the fact that there are 9 different sets of customers throughout the day. Each set of customers will use all the tables, so we need to multiply the estimated number of containers by the number of sets of customers to get an accurate estimation for the day.

Let's analyze the options provided:

A) 50 × 9: This option assumes there are 50 tables, which is incorrect based on the given information.

B) 16 × 3 × 9: This option assumes there are 16 tables, which is incorrect based on the given information.

C) 50 × 3 × 10: This option assumes there are 50 tables and 10 different sets of customers. Although the number of tables is incorrect, this option accounts for the number of salsa types and the number of sets of customers. However, it does not accurately represent the given scenario.

D) 40 × 5 × 5: This option assumes there are 40 tables and 5 different sets of customers. It also considers the number of salsa types. However, it does not accurately represent the given scenario as the number of tables is incorrect.

None of the options provided accurately represent the given scenario. The correct expression to estimate the number of containers of salsa needed for all the tables in one day would be:48 × 3 × 9

For more such question on expression visit:

https://brainly.com/question/1859113

#SPJ8

which is true regarding the sequence below?
5,2,-3,-10,-19

Answers

Answer: Its going down by odd numbers

Step-by-step explanation:

5-2=3, 2-(-3)= 5, then it would go on 7, 9, 11, 13...

Answer:

The difference between the numbers does not follow a common pattern; hence, the sequence is not arithmetic.

Step-by-step explanation:

Let's take the 1st two digits, 5 and 2

The difference between the numbers, 5-2 = 3

Following this pattern,

2-(-3) = 5

-3-(-10) = 7 and,

-10-(-19) = 9.

We can see that the differences are 3, 5, 7 and 9 and therefore we can prove that there is no common difference between the numbers.

Hence the sequence does not follow an arithmetic pattern as there is no common difference

To learn more about sequence

https://brainly.com/question/3615472

Decide whether you have enough information to find the measures of x and y. If you do, find the angle
If you don't, write "Not Enough Info" (NEI)

Answers

A. x=135* B. y+135*=180*. C. x=135*
y=135* y=180*-135*=45*. y=45*
x=(NEI)
D. x=45*
y=(NEI)

please help i dont know how to do this

Answers

Answer:

CD: 8.5

m<D: 20.6°

m<C: 69.4°

Step-by-step explanation:

CD:

The first thing it wants us to do is find the length of CD. Since the triangle shown is a right triangle, we can use Pythagorean theorem ([tex]a^{2}+b^{2}=c^{2}[/tex])  to solve for the missing length. It's important to remember that when using the Pythagorean theorem, c is the hypotenuse.

[tex]a^{2}+b^{2}=c^{2}\\3^{2}+8^{2}=c^{2}\\9 + 64 = c^{2}\\73 = c^{2}\\\sqrt{73} =c[/tex]

Since our answer is no an integer, we must turn it into a decimal.

[tex]\sqrt{73}[/tex] ≈ 8.544003745 ≈ 8.5

m<D:

Now, they want us to find the measure of <D. To do this, we will need to use trig functions (sine, cosine, tangent). To help us determine which trig function to use, we can remember the acronym SOH CAH TOA. This acronym tells us that sine is equal to opposite divided by hypotenuse, cosine is equal to adjacent divided by hypotenuse, and tangent is equal to opposite divided by adjacent. Since we do the hypotenuse and sides adjacent and opposite of <D, we can choose whichever trig function we want. For this problem, we will use tangent, so we can avoid using a rounded number, 8.5, as one of our sides.

Tan(D) = opposite / adjacent

Tan(D) = 3 / 8     [Take the tan inverse of both sides}

[tex]Tan^{-1}(Tan(D))=Tan^{-1}(3/8)[/tex]     [Simplify]

[tex]D=Tan^{-1}(3/8)[/tex]     [Solve]

D ≈ 20.55604522

D ≈ 20.6°

m<C:

Lastly, we must find the last unknown angle on the triangle. Since all angles on a triangle total 180°, if know that <C+<D+<E=180°. Let's solve this equation.

<C+<D+<E=180°

<C + 20.6 + 90 = 180     [Add]

<C + 110.6 = 180     [Subtract]

<C = 180 - 110.6     [Solve]

<C = 69.4°

If this answer helped you, please leave a thanks!

Have a GREAT day!!!

I need help , any of u guys have the answer?

Answers

the answer the third choice
(x to the power of 2+1)+(5-x)
first, u keep x to the power of two because there are no like terms, and then you combine the like terms 1 and 5 to get 6, then you subtract x

therefore, the answer is x squared minus x plus 6

I REALLY NEED HELP!!!! I need to get done by the end of June so I can Play on the COGL (crafters of God's love) server on M i n e c r a f t!!!
HELP!!!

A merchant uses the rule: selling price equals cost plus 10% of cost. This is a function. _-_-_Why?_-_-_
_-_-_Why?_-_-_
_-_-_Why?_-_-_
I don't need the equasion, I need to know how it's a function.

Answers

The rule used by the merchant is a function because it establishes a relationship between the cost of an item and its selling price

What is a function?

A function is simply described as a mathematical rule, law or expression that shows the relationship between two variables.

These two variables are known as;

The dependent variablesThe independent variables

From the information given, we have that;

Function assigns a unique selling price for every cost input

Input: cost

Output: selling price = cost + 10% of cost Formula: SP = C + 0.1C.

By following this rule, consistent selling prices can be determined efficiently based on cos

Learn more about functions at: https://brainly.com/question/11624077

#SPJ1

Please help!!!! I will give points to correct answer !!!

Answers

The equation that shows the Pythagorean identity is true for θ = 270° and is in the form sin²θ + cos²θ = 1 is option  B. 0² + (-1)² = 1

The Pythagorean identity is a fundamental trigonometric identity that relates the sine and cosine functions. It states that for any angle θ, the sum of the squares of the sine and cosine of that angle is equal to 1: sin²θ + cos²θ = 1.

We are given θ = 270° and we need to select the equation that satisfies the Pythagorean identity in the given form.

Let's evaluate each option:

A. 0² + 1² = 1

In this case, sin²θ = 0² = 0 and cos²θ = 1² = 1. Adding them together, we get 0 + 1 = 1, which satisfies the Pythagorean identity.

B. 0² + (−1)² = 1

Here, sin²θ = 0² = 0 and cos²θ = (−1)² = 1. Adding them, we have 0 + 1 = 1, which satisfies the Pythagorean identity.

C. (−1)² + 0² - 1

In this equation, sin²θ = (−1)² = 1 and co

s²θ = 0² = 0. However, the equation does not satisfy the Pythagorean identity because 1 + 0 - 1 ≠ 1.

D. 1² + 0² = 1

For this option, sin²θ = 1² = 1 and cos²θ = 0² = 0. Adding them together, we get 1 + 0 = 1, which satisfies the Pythagorean identity.

Based on our evaluation, options A and B both satisfy the Pythagorean identity for θ = 270°. Therefore, either A or B can be selected as the correct equation.The correct answer is  b.

Know more about   Pythagorean identity   here:

https://brainly.com/question/24287773

#SPJ8

The Question was Incomplete, Find the full content below :

Which equation shows that the Pythagorean identity is true for θ = 270°?

Select the equation that is in the form sin²θ+ cos²θ = 1.

A. 0² + 1² = 1

B. 0² + (−1)² = 1

C. (-1)² + 0² - 1

D. 1² + 0² = 1

= find the passible values of K if x² + (k-3) x+4 = 0​

Answers

The quadratic equation is x² + (k - 3)x + 4 = 0.

The values of k are 7 and -1.

Given: The quadratic equation is x² + (k - 3)x + 4 = 0.

Now, we can find the possible values of k.

To find the values of k, we will apply the discriminant formula of quadratic equation which is given by: [tex]$D=b^2-4ac$[/tex] ,where a,b and c are the coefficients of the quadratic equation: ax²+bx+c

Roots of the quadratic equation are given by:

[tex]$x=\frac{-b\pm \sqrt{b^2-4ac}}{2a}$[/tex]

Now, let's apply these formulas to the given quadratic equation:

x² + (k - 3)x + 4 = 0

Comparing with the standard quadratic equation of the form ax² + bx + c = 0, we get:

[tex]a = 1, b = k - 3, and c = 4$\\D = b^2 - 4ac$= $(k - 3)^2 - 4(1)(4)$= $k^2 - 6k + 9 - 16$= $k^2 - 6k - 7$[/tex]

The roots of the given quadratic equation are given by:

[tex]$x = \frac{-b \pm \sqrt{b^2 - 4ac}}{2a}$[/tex]

Substituting the values of a, b, c, and D, we get:

[tex]$x = \frac{-(k - 3) \pm \sqrt{(k - 3)^2 - 4(1)(4)}}{2(1)}$$x = \frac{3 - k \pm \sqrt{k^2 - 6k - 7}}{2}$[/tex]

Now, for the quadratic equation to have real and equal roots, the discriminant must be equal to zero, i.e., [tex]$D = 0$$\ therefore, k^2 - 6k - 7 = 0$.[/tex]

Factoring the quadratic equation, we get:

[tex]$k^2 - 7k + k - 7 = 0$$\\k(k - 7) + 1(k - 7) = 0$$\\(k - 7)(k + 1) = 0$[/tex]

So, the possible values of k are k = 7 and k = -1.

Hence, the values of k are 7 and -1.

For more question on equation

https://brainly.com/question/17145398

#SPJ8

If a school room is 32 metres long and 11 metres wide, how many boy will it accommodate allowing 8sq. metres to each boy? A. 56 boys B. 54 boys C. 44 boys D. 36 boys E. 34 boys​

Answers

Answer:

[tex]\huge\boxed{\sf 44\ boys}[/tex]

Step-by-step explanation:

Area of rectangle:

= Length × Width

Given data:

Length = 32 m

Width = 11 m

Solution:

Area of school room:

= Length × Width

= 32 × 11

= 352 m²

Number of boys to accommodate 8 sq. meters:

= 352 / 8

= 44 boys

[tex]\rule[225]{225}{2}[/tex]

In a large restaurant, there are 9 times as many chairs as tables. The restaurant is famous for its very spicy chili. If the restaurant has 360 chairs, how many tables are in the restaurant?

Answers

Answer:

There are 40 tables.

Step-by-step explanation:

Since we know that there are 9 times as many chairs, then there are tables, all we have to do is divide the number of chairs by 9, and we get the answer 40.

A car that averages 22 miles per gallon emits 4.3 metric tons of carbon dioxide per year. A passenger bus emits 9.2 metric tons of carbon dioxide per year and can carry 30 people at a time. How much less carbon dioxide does a commuter who takes a bus emit in a year compared to a commuter who drives everyday?
13.5 metric tons
2.1 metric tons
5.1 metric tons
3.99 metric tons

Answers

The answer is 3.99 metric tons.

To calculate the difference in carbon dioxide emissions between a commuter who takes a bus and one who drives a car every day, we need to compare the emissions of each mode of transportation per person.

The car averages 22 miles per gallon, which means it emits 4.3 metric tons of carbon dioxide per year. However, we don't know the number of passengers in the car.

The passenger bus emits 9.2 metric tons of carbon dioxide per year and can carry 30 people at a time. To find the emissions per person, we divide the total emissions by the number of passengers. In this case, each person on the bus emits 9.2 / 30 = 0.3067 metric tons of carbon dioxide per year.

To determine the difference in emissions, we subtract the emissions per person for the bus from the emissions per person for the car. Therefore, the difference is 4.3 - 0.3067 = 3.9933 metric tons of carbon dioxide per year.

Rounding this value to two decimal places, we get 3.99 metric tons.

Therefore, the answer is 3.99 metric tons.

To learn more on average:
https://brainly.com/question/130657
#SPJ8

What is the domain of the square root function graphed below?

On a coordinate plane, a curve open up to the right in quadrant 4. It starts at (0, negative 1) and goes through (1, negative 2) and (4, negative 3).
x less-than-or-equal-to negative 1
x greater-than-or-equal-to negative 1
x less-than-or-equal-to 0
x greater-than-or-equal-to 0
Mark this and return

Answers

The domain of the square root function is x greater-than-or-equal-to 0, since the function is defined for all non-negative x-values or x-values greater than or equal to zero.

The domain of the square root function graphed below can be determined by looking at the x-values of the points on the graph.

From the given information, we can see that the curve starts at (0, -1) and goes through (1, -2) and (4, -3).

The x-values of these points are 0, 1, and 4.

Since the square root function is defined for any non-negative x-values or x-values more than or equal to zero, its domain is x greater-than-or-equal-to 0.

for such more question on domain

https://brainly.com/question/16444481

#SPJ8

six people want equally share 1 1/2 pizzas. how much pizza does each person get?

Answers

Each person gets 4 slices of pizza.

Given there are 6 people who want to equally share 1 1/2 pizzas, we can set up an equation by first converting 1 1/2 to an improper fraction:

1 1/2 = ((2 • 1) + 1) / 2
1 1/2 = 3/2

Now, we can divide 6 by 3/2:

6 / (3/2) = (6 • 2) / 3
12 / 3 = 4

Therefore, 6 / 1 1/2 = 4. This means each person receives 4 slices of pizza.

Outside temperature over a day can be modelled as a sinusoidal function. Suppose you know the high temperature for the day is 80 degrees and the low temperature of 50 degrees occurs at 5 AM. Assuming t is the number of hours since midnight, find an equation for the temperature, D, in terms of t.

Answers

To find an equation for the temperature, D, in terms of t, we can use the properties of a sinusoidal function to model the temperature variation over the day.

Given:

High temperature: 80 degrees

Low temperature occurs at 5 AM (t = 5)

t is the number of hours since midnight

Let's assume a sinusoidal function of the form:

D = A * sin(B * t + C) + Dc

where:

A represents the amplitude (half the difference between the high and low temperatures)

B represents the frequency (how many cycles occur over a 24-hour period)

C represents the phase shift (how much the function is shifted horizontally)

Dc represents the vertical shift (the average temperature throughout the day)

We can determine the values of A, B, C, and Dc based on the given information.

Amplitude (A):

The amplitude is half the difference between the high and low temperatures:

A = (80 - 50) / 2

= 30 / 2

= 15 degrees

Frequency (B):

Since we want the temperature to complete one cycle over a 24-hour period, the frequency can be calculated as:

B = 2π / 24

Phase Shift (C):

Since the low temperature occurs at 5 AM (t = 5), the function should be shifted horizontally by 5 hours. To convert this to radians, we multiply by (2π / 24):

C = 5 * (2π / 24)

Vertical Shift (Dc):

The average temperature throughout the day is the midpoint between the high and low temperatures:

Dc = (80 + 50) / 2

= 130 / 2

= 65 degrees

Now we can put all the values together to obtain the equation for the temperature, D, in terms of t:

D = 15 * sin((2π / 24) * t + (5 * 2π / 24)) + 65

Simplifying further:

D = 15 * sin((π / 12) * t + (π / 12)) + 65

Therefore, the equation for the temperature, D, in terms of t is:

D = 15 * sin((π / 12) * t + (π / 12)) + 65.

Using the Empirical Rule, approximate the following percentages for Parts A - E.
The distribution of weights of newborn babies in one region is bell-shaped with a mean of 3000 grams and standard deviation of 500 grams. For all questions below, show all relevant work.


Part A :Approximately, what percentage of newborn babies weigh more than 3000 grams?
Part B : Approximately, what percentage of newborn babies weigh more than 2000 grams?
Part C : Approximately, what percentage of newborn babies weigh less than 4000 grams?
Part D : Approximately, what percentage of newborn babies weigh between 2000 and 4000 grams?
Part E : What is the range of birth weights that would contain the middle 68% of newborn babies' weights?

Answers

Part A: Approximately 50% of newborn babies weigh more than 3000 grams. Part B: Approximately 84.13% of newborn babies weigh more than 2000 grams. Part C: Approximately 84.13% of newborn babies weigh less than 4000 grams. Part D: Approximately 68% of newborn babies weigh between 2000 and 4000 grams. Part E: The range of birth weights that would contain the middle 68% of newborn babies' weights is from 2500 grams to 3500 grams.

1: Calculate the Z-scores for the given weights using the formula: Z = (X - μ) / σ, where X is the value, μ is the mean, and σ is the standard deviation.

For Part A:

Z = (3000 - 3000) / 500 = 0

Using the Z-table, we find that the percentage of babies weighing more than 3000 grams is approximately 50%.

For Part B:

Z = (2000 - 3000) / 500 = -2

Using the Z-table, we find that the percentage of babies weighing more than 2000 grams is approximately 97.72%. Since we want the percentage of babies weighing more than 2000 grams, we subtract this value from 100% to get approximately 2.28%.

For Part C:

Z = (4000 - 3000) / 500 = 2

Using the Z-table, we find that the percentage of babies weighing less than 4000 grams is approximately 97.72%.

For Part D:

To find the percentage of babies weighing between 2000 and 4000 grams, we subtract the percentage of babies weighing more than 2000 grams from the percentage of babies weighing less than 4000 grams.

Approximately 97.72% - 2.28% = 95.44%

For Part E:

Since the Empirical Rule states that approximately 68% of the data falls within one standard deviation of the mean, we need to find the weights that correspond to the boundaries of this range.

The lower boundary would be the mean minus one standard deviation: 3000 - 500 = 2500 grams.

The upper boundary would be the mean plus one standard deviation: 3000 + 500 = 3500 grams.

Therefore, the range of birth weights that would contain the middle 68% of newborn babies' weights is from 2500 grams to 3500 grams.

For more such questions on weigh, click on:

https://brainly.com/question/29892643

#SPJ8

PLEASE ANSWER AND FILL IN THE BOX IF NEEDED

Answers

Answer:

  A.  p(t) = 16 -5t +2t²

Step-by-step explanation:

You want the interpolating polynomial for points (1, 13), (2, 14), and (3, 19).

Quadratic regression

The given system of equations can be solved by your favorite method to find ...

a₀ = 16a₁ = -5a₂ = 2

The interpolating polynomial is p(t) = 16 -5t +2t².

__

Additional comment

The points and the polynomial are graphed in the second attachment.

<95141404393>

The interpolating polynomial  is (a) p(t) = 13 - 2.5t +1.5t²

How to determine the interpolating polynomial

From the question, we have the following parameters that can be used in our computation:

Points =  (1, 13), (2, 14), and (3, 19).

Using the general form of the quadratic formula, we have

a + b + c = 12

4a + 2b + c = 14

9a + 3b + c = 19

Subtract the equations to eliminate c

So, we have

3a + b = 2

5a + b = 5

Subtract the equations to eliminate b

So, we have

2a = 3

So, we have

a = 1.5

Next, we have

3a + b = 2

3(1.5) + b = 2

Evaluate

b = -2.5

Lastly, we have

4a + 2b + c = 14

4(1.5) + 2(-2.5) + c = 14

Evaluate

c = 13

So, the equation is p(t) = 13 - 2.5t +1.5t²

Read more about polynomial at

https://brainly.com/question/30833611

#SPJ1

Given rhombus LMNO below. If m

Answers

The measure of angle LOP is given as follows:

m < LOP = 52º.

How to obtain the angle measure?

The segment OP bisects the angle O of the rhombus into two smaller angles, which are NOP and LOP.

.

A bisection means that the larger angle is divided into two smaller angles of equal measure.

The measure of angle NOP is given as follows:

m < NOP = 52º.

Hence the measure of angle LOP is given as follows:

m < LOP = 52º.

Missing Information

The final sentence is to find the measure of angle LOP.

More can be learned about angle measures at brainly.com/question/25215131

#SPJ1

Simplify the expression so there is only one positive power for the base, -5.

Answers

Answer:

c

Step-by-step explanation:

it's a property of powers, when the base is the same (-5) , you need to

sum the powers when both terms are multiplyng

subtract the powers when both terms are dividing (numerator power minus denominator power, in that order)

3. In ∆ JAM, which of the following statement is always TRUE?

Answers

The option that shows the missing angles in the triangle is:

Option C: m∠1 < m∠4

How to identify the missing angle?

We know that the sum of angles in a triangle is 180 degrees.

Therefore looking at the given triangle, we can say that:

m∠1 + m∠2 + m∠3 = 180°

We also know that the sum of angles on a straight line is 180 degrees and as such we can say that:

m∠3 +  m∠4 = 180°

By substitution we can say that:

m∠4 = m∠1 + m∠2

Thus:

m∠1 < m∠4

Read more about Missing Angle at: https://brainly.com/question/28293784

#SPJ1

The missing options are:

m∠1 > m∠4

m∠2 > m∠4

m∠1 < m∠4

m∠3 = m∠4

Please help!! I don’t understand what to do

Answers

Answer:

y = x^2 + 6x +7

y=-x+1

Step-by-step explanation:

To answer this question, we need to find the equation for the line and for the parabola. So, let's do just that!

Line:

The line has a slope of -1 and and y-intercept of 1. This means that the equation for the line is y=-x+1.

Parabola:

To find the vertex form of a parabola, we must first find the vertex form of a parabola. The vertex form of a parabola is y=a(x-h)^2+k, where the vertex is (h,k). Since the vertex of the parabola is (-3,-2), h=-3 and k=-2. Let's plug these values into the vertex form of a parabola.

y=a(x-h)^2+k

y=a(x-(-3))^2+(-2)

y=a(x+3)^2-2

We're not done yet, though, as we still need to find the value of a. To do this, we will take one of the points on the parabola and plug it into the equation. I will be using the point (-1,2).

y=a(x+3)^2-2     [Plug in x and y values]

2=a((-1)+3)^2-2     [Simplify]

2=a(2)^2-2     [Simplify]

2=4a-2     [Add 2 to both sides]

4=4a     [Divide both sides by 4]

a=1

Now, we know that the equation of our parabola in vertex form is y=(x+3)^2-2. This isn't what the problem is asking for, though. Instead, they want the standard form of the parabola. To do this, we will need to expand y=(x+3)^2-2.

y=(x+3)^2-2

y=x^2 + 6x + 9 -2

y = x^2 + 6x +7

If this answer helped you, please leave a thanks!

Have a GREAT day!!!

F(x) = 2x + 3 and g(x) = -4x - 27 find the intersection

Answers

Answer: (-5, -7)

Step-by-step explanation: To find the intersection of two functions, you need to set them equal to each other and solve for x.

In this case, we have:

2x + 3 = -4x - 27

Adding 4x to both sides, we get:

6x + 3 = -27

Subtracting 3 from both sides, we get:

6x = -30

Dividing both sides by 6, we get:

x = -5

Now that we have the value of x, we can find the corresponding value of y by plugging it into either of the original equations. Let's use f(x) = 2x + 3:

f(-5) = 2(-5) + 3 = -7

Therefore, the intersection of the two functions is (-5, -7).

he class is trying to determine who will take care of the class hamster for the weekend. In order to win the chance, you must flip heads on a coin and then spin an even number on a spinner with 9 equal sections labeled from 1 - 9. What is the probability of winning the chance to take care of the hamster?

Answers

Answer:

2/9

Step-by-step explanation:

Coin

A coin has 1 side heads and 1 side tails.

total number of possible outcomes = 2

desired outcome: heads

number of desired outcomes = 1

p(event) = (number of desired outcomes)/(total number of possible outcomes)

p(heads) = 1/2

Spinner

The spinner has 9 sections of equal size.

possible outcomes: the numbers 1, 2, 3, 4, 5, 6, 7, 8, 9

total number of possible outcomes = 9

desired outcome: even number

number of desired outcomes = 4

p(event) = (number of desired outcomes)/(total number of possible outcomes)

p(even number) = 4/9

Combined probability

The coin and spinner are independent events. The overall probability of independent events is the product of the individual probabilities.

p(heads then even) = 1/2 × 4/9 = 4/18 = 2/9

Answer: 2/9

Which graph best represents the solution to the following pair of equations?

y = 4x + 2
y = x + 5

A graph is plotted with values ranging from negative 10 to 10 on both x axis and y axis at increments of 1. Two lines having equations y is equal to 4 times x plus 2 and y is equal to x plus 5 are plotted. These 2 lines intersect at the ordered pair 1, 6.
A graph is plotted with values ranging from negative 10 to 10 on both x axis and y axis at increments of 1. Two lines having equations y is equal to 4 times x plus 2 and y is equal to x plus 5 are plotted. These 2 lines intersect at the ordered pair 2, negative7.
A graph is plotted with values ranging from negative 10 to 10 on both x axis and y axis at increments of 1. Two lines having equations y is equal to 4 times x plus 2 and y is equal to x plus 5 are plotted. These 2 lines intersect at the ordered pair negative 1, negative 6.
A graph is plotted with values ranging from negative 10 to 10 on both x axis and y axis at increments of 1. Two lines having equations y is equal to 4 times x plus 2 and y is equal to x plus 5 are plotted. These 2 lines intersect at the ordered pair negative 2, 7.

Answers

The given pair of equations is y = 4x + 2 and y = x + 5, and we are to determine which of the given graphs represents their solution. The first equation is in slope-intercept form, y = mx + b, where m is the slope and b is the y-intercept. Comparing this with the given equation, we see that its slope is 4 and y-intercept is 2.

The second equation is also in slope-intercept form, y = mx + b. Comparing it with the given equation, we see that its slope is 1 and y-intercept is 5.Since we have two lines, we need to find their point of intersection. Substituting y = 4x + 2 into y = x + 5, we have4x + 2 = x + 5Simplifying the equation, we get3x = 3, which gives x = 1.

Substituting this value of x into either of the equations, say y = 4x + 2, we have y = 4(1) + 2 = 6. Hence, the point of intersection is (1, 6). Now, let's examine the given graphs and see which one has (1, 6) as a point of intersection:

Graph 1: The line y = 4x + 2 passes through (0, 2) and has a slope of 4, which means it will be steeper than the line y = x + 5. The line y = x + 5 passes through (0, 5) and has a slope of 1, which means it will be flatter than the line y = 4x + 2. Hence, these two lines cannot intersect at (1, 6). Graph 1 is not the solution.

Graph 2: The line y = 4x + 2 passes through (-1, -2) and has a slope of 4, which means it will be steeper than the line y = x + 5. The line y = x + 5 passes through (0, 5) and has a slope of 1, which means it will be flatter than the line y = 4x + 2. Hence, these two lines cannot intersect at (1, 6). Graph 2 is not the solution.

Graph 3: The line y = 4x + 2 passes through (-1, -2) and has a slope of 4, which means it will be steeper than the line y = x + 5. The line y = x + 5 passes through (0, 5) and has a slope of 1, which means it will be flatter than the line y = 4x + 2.

Hence, these two lines cannot intersect at (1, 6). Graph 3 is not the solution.Graph 4: The line y = 4x + 2 passes through (0, 2) and has a slope of 4, which means it will be steeper than the line y = x + 5. The line y = x + 5 passes through (0, 5) and has a slope of 1, which means it will be flatter than the line y = 4x + 2. Hence, these two lines cannot intersect at (1, 6). Graph 4 is not the solution.

Therefore, none of the given graphs represents the solution to the pair of equations.

For more such questions on slope-intercept

https://brainly.com/question/1884491

#SPJ8

Other Questions
Novak Compaty'snet income for 2020 it 5641,000 , and 79.000 shares of commcenstock were issued and outstandine during 2020 The onliv potentialy dilutive teciarities outstandeng were 27000 encoutive stock options iswed during 2019 , each exreisable for one share at $19.50, none of these have been exercised. The overape market price of Norak's stock during 2020 was $2500, (a) Compute diluted eaminci per share (Round answer to 2 decimal places, e. a..55) Diluted eaenings per share $ _____ (b) Ascume the same facts as those assumed for part lah, eveept that 10000 additional ootiont were issied on Octoter 1 . 2020 with 2020 war 52850 (Alound anwer to 2 derimaf places, es 2.55). Diluted eranings per share $ _____ In many jurisdictions there are laws governing the lowestpermissible wage to be paid to a worker. Do these rules impact allworkers and all employers? Support your answer withgraph(s). Since the Hackman and Oldham model was developed in the 1970s, jobs have changed in what way?a) increased in turnover and job satisfactionb) increased in autonomy and skill varietyc) decreased in motivation and satisfactiond) decreased in task identify and responsibility Post a brief description of your topic of research interest. Next, state the philosophical orientation that reflects your worldview and explain the epistemological and ontological assumptions of this orientation. Then, explain how these assumptions lend themselves to one or more research approaches. (X) + (E^X)Y'(X) + Xy(X) = Cos(X)Determine The Particular Solution Up To Terms Of Order O(X^5) In Its Power Series Representation About X=0y''(x) + (e^x)y'(x) + xy(x) = cos(x)Determine the particular solution up to terms of order O(x^5) in its power series representation about x=0 What is symbolism in the Iliad and the odyssey by Homer ?? This problem is an example of critically damped harmonic motion. A mass m = 8 kg is attached to both a spring with spring constant k = 392 N/m and a dash-pot with damping constant c = 112 N. s/m. The ball is started in motion with initial position xo = 9 m and initial velocity vo = -64 m/s. Determine the position function (t) in meters. x(t) le Graph the function x(t). Now assume the mass is set in motion with the same initial position and velocity, but with the dashpot disconnected (so c = 0). Solve the resulting differential equation to find the position function u(t). In this case the position function u(t) can be written as u(t) = Cocos(wotao). Determine Co, wo and a. Co = le Wo O (assume 0 0 < 2) Finally, graph both function (t) and u(t) in the same window to illustrate the effect of damping. The order of convergence for finding one of the roots of f(x) = x(1 cosx) =0 using Newtons method is (Hint: P=0): Select one: O a=1 a = 2 a = 3 O= 4 which windows component prompts the user for credentials or permission Babosa Freight Inc. is seeking to raise financing for the construction of a new freight terminal beginning January 1, 2018. The construction cost of the freight terminal is estimated at $20 million. You have been asked to prepare a report for the companys Board of Directors to evaluate the best financing arrangement under different scenarios. You have narrowed down your choices to the following alternatives: Alternative 1: Raise the required amount from the proceeds of a new 6% coupon bond with a face value of $ 21,764,514.48, and a maturity period of 5 years. The annual market interest rate is 8%. The coupon payment is payable semiannually. Alternative 2: A private equity firm has offered to finance the entire construction in a financing arrangement whereby Babosa Freight Inc. would make ten equal semiannual installment payments of exactly $2,465,817.61 each for five years. The appropriate annual market interest rate implied in the arrangement is 8%. Required: Round answers to the nearest whole dollar Please use the provided PV tables. Determine the annual interest expense for the year ending December 31, 2018 for each e financing alternative. Which financing alternative would you recommend to Babosa Freights Board of Directors if the companys objective is to show the lowest reported long term debt liability on its balance sheet for the year ended December 31st 2018? Suppose the solution to the differential equation (x - 3)y" + 3y = 0 is written as a power series y = = a, (x-1)" What is the lower bound of the radius of convergence of 71-0 this power series? a) 0.5 c)2 d)3 e) [infinity] b)1 6) If a series solution is to be found for y"-4xy'+4y=0, y(0)=2, y'(0)=3 then a2 = (a) -4 (b) 8 (c) -8 (d) 1 e) NOTA 7) The lower bound for the radius of convergence for the series solution of (1+x)y"-xy'+3y=0 , Xo = 3 is 4 a) 4 b)-4 c) -1 e) NOTA d) 1 9) The exponents at the singularity for (x-1) y "+3x (x-1)y -3y = 0 are: (a) 1,-3 (b) 2,-3 (c) 3,-1 (d) 1,-2 10) For the equation x2y "+axy + y = 0, the values of a, so that the solutions approach zero as x 0: a) a 0 e) NOTA e) NOTA based on the description above, what kind of magazine is national geographic? A is a phenomenon in which the form of return, contrary to theefficient market hypothesis, continues to appear.What is A? find parametric equations and symmetric equations for the line of intersection of the planes Sand falls from an overhead bin and accumulates in a conical pile with a radius that is always three times its height. Suppose the height of the pile increases at a rate of 2 cm/s when the pile is 12 cm high. At what rate is the sand leaving the bin at that instant? 1 (note: the volume of a cone is V = rh) It's a good idea to take extra copies of your rsum with you to an interview, as well as a list of questions to ask and any past correspondence about the position. True False Product testing for reliability and quality helps to ensure a consumer's right toa) be heard.b) be informed.c) choose.d) performance.e) safety. f +n+1- ff - nf2 - 2nPn 127P = 0.reduce this equation to first order systemthen solve the linear system by the block tridiagonal elimination techniquen=0.01assum any value you need. 3 2.85 points eBook Print References Sun Bank USA has purchased a 16 million one-year Australian dollar loan that pays 12 percent interest annually. The spot rate of U.S. dollars for Australian dollars (AUD/USD) is $0.757/A$1. It has funded this loan by accepting a British pound (BP)-denominated deposit for the equivalent amount and maturity at an annual rate of 10 percent. The current spot rate of U.S. dollars for British pounds (GBP/USD) is $1.320/1. a. What is the net interest income earned in dollars on this one-year transaction if the spot rate of U.S. dollars for Australian dollars and U.S. dollars for BPs at the end of the year are $0.715/A$1 and $1.520/1, respectively? (Negative amount should be indicated by a minus sign. Do not round intermediate calculations. Enter your answers in dollars, rather than in millions of dollars. Round your final answer to the nearest whole dollar. (e.g., 32)) b. What should the spot rate of U.S. dollars for BPs be at the end of the year in order for the bank to earn a net interest income of $200,000 (disregarding any change in principal values)? (Round your answer to 5 decimal places. (e.g., 32.16161)) a. b. Check my work Net interest income Spot rate of U.S. dollars $ 59 Determine the intervals on which each of the following functions is continuous. Show your work. (1) f(x)= x-x-2 x-2 1+x (2) f(x)=2-x x 0 0< x2 (x-1) x>2